Skip to main content

Existence of periodic solutions for a class of second order Hamiltonian systems

Abstract

By using the least action principle and the minimax methods, the existence of periodic solutions for a class of second order Hamiltonian systems is considered. The results obtained in this paper extend some previous results.

1 Introduction and main results

Consider the second order Hamiltonian system

$$ \textstyle\begin{cases} \ddot{u}(t)=\nabla F(t,u(t)),\\ u(T)-u(0)=\dot{u}(T)-\dot{u}(0)=0, \end{cases} $$
(1.1)

where \(T>0\) and \(F:[0,T]\times\mathbb{R}^{N}\rightarrow\mathbb{R}\) satisfies the following assumption:

(A) \(F(t,x)\) is measurable in t for every \(x\in\mathbb{R}^{N}\), continuously differentiable in x for a.e. \(t\in[0,T]\), and there exist \(a\in C(\mathbb{R}^{+},\mathbb{R}^{+})\), \(b\in L^{1}([0,T];\mathbb {R}^{+})\) such that

$$\bigl\vert F(t,x)\bigr\vert \leq a\bigl(\vert x\vert \bigr)b(t), \qquad \bigl\vert \nabla F(t,x)\bigr\vert \leq a\bigl(\vert x\vert \bigr)b(t) $$

for all \(x\in\mathbb{R}^{N}\) and a.e. \(t\in[0,T]\).

The corresponding functional \(\varphi:H_{T}^{1}\rightarrow\mathbb {R}\),

$$\varphi(u)=\frac{1}{2}\int_{0}^{T} \bigl\vert \dot{u}(t)\bigr\vert ^{2}\,dt+\int_{0}^{T}F \bigl(t,u(t) \bigr)\,dt, $$

is continuously differentiable and weakly lower semi-continuous on \(H_{T}^{1}\), where \(H_{T}^{1}\) is the usual Sobolev space with the norm

$$\Vert u\Vert = \biggl(\int_{0}^{T} \bigl\vert u(t)\bigr\vert ^{2} \,dt+\int_{0}^{T} \bigl\vert \dot {u}(t)\bigr\vert ^{2}\,dt \biggr)^{1/2} $$

for \(u\in H^{1}_{T}\), and

$$\bigl\langle \varphi'(u),v \bigr\rangle =\int ^{T}_{0} \bigl[ \bigl(\dot{u}(t),\dot{v} \bigr)+ \bigl( \nabla F \bigl(t,u(t) \bigr),v(t) \bigr) \bigr]\,dt $$

for all \(u, v\in H^{1}_{T}\), It is well known that the solutions of problem (1.1) correspond to the critical points of φ.

The existence of periodic solutions for problem (1.1) is obtained in [122] with many solvability conditions by using the least action principle and the minimax methods, such as the coercive type potential condition (see [2]), the convex type potential condition (see [5]), the periodic type potential conditions (see [16]), the even type potential condition (see [4]), the subquadratic potential condition in Rabinowitz’s sense (see [9]), the bounded nonlinearity condition (see [6]), the subadditive condition (see [11]), the sublinear nonlinearity condition (see [3, 13]), and the linear nonlinearity condition (see [7, 15, 19, 20]).

In particular, when the nonlinearity \(\triangledown F(t,x)\) is bounded, that is, there exists \(g(t)\in L^{1}([0,T],\mathbb{R}^{+})\) such that \(\vert \triangledown F(t,x)\vert \leq g(t)\) for all \(x\in\mathbb{R}^{N}\) and a.e. \(t\in[0,T]\), and that

$$\int_{0}^{T}F(t,x)\,dt\rightarrow\pm \infty \quad \mbox{as } \vert x\vert \rightarrow\infty, $$

Mawhin and Willem [6] proved that problem (1.1) has at least one periodic solution.

In [3, 13], Han and Tang generalized these results to the sublinear case:

$$ \bigl\vert \triangledown F(t,x)\bigr\vert \leq f(t)\vert x\vert ^{\alpha}+g(t) \quad \text{for all } x\in \mathbb{R}^{N} \text{ and a.e. } t\in[0,T] $$
(1.2)

and

$$ \vert x\vert ^{-2\alpha}\int_{0}^{T}F(t,x) \,dt \rightarrow\pm\infty \quad \text{as } \vert x\vert \rightarrow\infty, $$
(1.3)

where \(f(t),g(t)\in L^{1}([0,T],\mathbb{R}^{+})\) and \(\alpha\in[0,1)\).

Subsequently, when \(\alpha=1\) Zhao and Wu [19, 20] and Meng and Tang [7, 15] also proved the existence of periodic solutions for problem (1.1), i.e. \(\nabla F(t,x)\) was linear:

$$\bigl\vert \triangledown F(t,x)\bigr\vert \leq f(t)\vert x \vert +g(t) \quad \mbox{for all }x\in \mathbb{R}^{N} \text{ and a.e. } t\in[0,T], $$

where \(f(t),g(t)\in L^{1}([0,T],\mathbb{R}^{+})\).

Recently, Wang and Zhang [21] used a control function \(h(\vert x\vert )\) instead of \(\vert x\vert ^{\alpha}\) in (1.2) and (1.3) and got some new results, where h satisfied the following conditions:

(B) \(h\in C([0,\infty),[0,\infty))\) and there exist constants \(C_{0}>0\), \(K_{1}>0\), \(K_{2}>0\), \(\alpha\in[0,1)\) such that

  1. (i)

    \(h(s)\leq h(t) \) \(\forall s\leq t\), \(s, t\in[0,\infty)\),

  2. (ii)

    \(h(s+t)\leq C_{0}(h(s)+h(t))\) \(\forall s, t\in[0,\infty)\),

  3. (iii)

    \(0\leq h(s)\leq K_{1}s^{\alpha}+K_{2}\) \(\forall s\in[0,\infty )\),

  4. (iv)

    \(h(s)\rightarrow\infty \) as \(s\rightarrow\infty\).

Motivated by the results mentioned above, we will consider the periodic solutions for problem (1.1). The following are our main results.

Theorem 1.1

Suppose that \(F(t,x)=F_{1}(t,x)+F_{2}(x)\), where \(F_{1}\) and \(F_{2}\) satisfy assumption (A) and the following conditions:

  1. (1)

    there exist \(f,g\in L^{1}([0,T];\mathbb{R}^{+})\) such that

    $$\bigl\vert \nabla F_{1}(t,x)\bigr\vert \leq f(t)h\bigl( \vert x\vert \bigr)+g(t), $$

    for all \(x\in\mathbb{R}^{N}\) and a.e. \(t\in[0,T]\), here h satisfies (B);

  2. (2)

    there exist constants \(r>0\) and \(\gamma\in[0,2)\) such that

    $$\bigl(\nabla F_{2}(x)-\nabla F_{2}(y),x-y \bigr) \geq-r\vert x-y\vert ^{\gamma}, $$

    for all \(x,y \in\mathbb{R}^{N}\);

  3. (3)
    $$\liminf_{\vert x\vert \rightarrow\infty}h^{-2}\bigl(\vert x\vert \bigr)\int_{0}^{T}F(t,x)\,dt> \frac {T^{2}C_{0}^{2}}{8\pi^{2}}\int _{0}^{T}f^{2}(t)\,dt. $$

Then problem (1.1) has at least one periodic solution which minimizes φ on \(H_{T}^{1}\).

Theorem 1.2

Suppose that \(F(t,x)=F_{1}(t,x)+F_{2}(x)\), where \(F_{1}\) and \(F_{2}\) satisfy assumption (A), (1), (2), and the following conditions:

  1. (4)

    there exist \(\delta\in[0,2)\) and \(\mu>0\) such that

    $$\bigl(\nabla F_{2}(x)-\nabla F_{2}(y),x-y \bigr) \leq \mu \vert x-y\vert ^{\delta}, $$

    for all \(x,y \in R^{N}\);

  2. (5)
    $$\limsup_{\vert x\vert \rightarrow\infty}h^{-2}\bigl(\vert x\vert \bigr)\int_{0}^{T}F(t,x)\,dt< -\frac {3T^{2}C_{0}^{2}}{8\pi^{2}} \int_{0}^{T}f^{2}(t)\,dt. $$

Then problem (1.1) has at least one periodic solution which minimizes φ on \(H_{T}^{1}\).

Theorem 1.3

Suppose that \(F(t,x)=F_{1}(t,x)+F_{2}(x)\), where \(F_{1}\) and \(F_{2}\) satisfy assumption (A), (1), and the following conditions:

  1. (6)

    there exists a constant \(0< r<4\pi^{2}/T^{2}\), such that

    $$\bigl(\nabla F_{2}(x)-\nabla F_{2}(y),x-y \bigr) \geq-r\vert x-y\vert ^{2}, $$

    for all \(x,y \in R^{N}\);

  2. (7)
    $$\liminf_{\vert x\vert \rightarrow\infty}h^{-2}\bigl(\vert x\vert \bigr)\int_{0}^{T}F(t,x)\,dt> \frac {T^{2}}{2(4\pi^{2}-rT^{2})}\int _{0}^{T}f^{2}(t)\,dt. $$

Then problem (1.1) has at least one periodic solution which minimizes φ on \(H_{T}^{1}\).

Theorem 1.4

Suppose that \(F=F_{1}+F_{2}\), where \(F_{1}\) and \(F_{2}\) satisfy assumption (A), (1), and the following conditions:

  1. (8)

    there exist \(k\in L^{1}([0,T];\mathbb{R}^{+})\) and (\(\lambda,\mu \))-subconvex potential \(G: \mathbb{R}^{N}\rightarrow\mathbb{R}\) with \(\lambda>1/2\) and \(0<\mu<2\lambda^{2}\), such that

    $$\bigl(\nabla F_{2}(t,x),y \bigr)\geq-k(t)G(x-y), $$

    for all \(x,y \in\mathbb{R}^{N}\);

  2. (9)
    $$\begin{aligned}& \limsup_{\vert x\vert \rightarrow\infty} h^{-2}\bigl(\vert x\vert \bigr)\int_{0}^{T}F_{1}(t,x)\,dt < - \frac{3T^{2}C_{0}^{2}}{8\pi^{2}}\int_{0}^{T}f^{2}(t) \,dt, \\& \limsup_{\vert x\vert \rightarrow\infty} \vert x\vert ^{-\beta}\int _{0}^{T}F_{2}(t,x) \,dt \leq-8\mu\max _{\vert s\vert \leq1}G(s)\int_{0}^{T}k(t) \,dt, \end{aligned}$$

    where \(\beta=\log_{2\lambda}(2\mu)\).

Then problem (1.1) has at least one periodic solution which minimizes φ on \(H_{T}^{1}\).

Remark 1.5

Theorems 1.1-1.4 extend some existing results: (i) [22], Theorems 1.1-1.4, are special cases of Theorems 1.1-1.4 with control function \(h(t)=t^{\alpha}, \alpha\in[0,1)\), \(t\in[0,+\infty)\); (ii) if \(F_{2}=0\), [15], Theorems 1 and 2, are special cases of Theorem 1.1 and Theorem 1.2, respectively; (iii) If \(F_{2}=0\), Theorem 1.1 and Theorem 1.2 extend [21], Theorems 1.1 and 1.2, since we weaken the so-called Ahmad-Lazer-Paul type conditions with the control function \(h(t)\).

2 Proof of theorems

For \(u\in H_{T}^{1}\), let \(\bar{u}=\frac{1}{T}\int_{0}^{T}\vert \dot {u}(t)\vert \,dt\) and \(\tilde{u}(t)=u(t)-\bar{u}\). Then one has

$$\begin{aligned}& \Vert \tilde{u}\Vert _{\infty}^{2} \leq\frac{T}{12}\int _{0}^{T}\bigl\vert \dot{u}(t)\bigr\vert ^{2}\,dt \quad \text{(Sobolev's inequality),} \\& \Vert \tilde{u}\Vert _{L^{2}}^{2} \leq\frac{T^{2}}{4\pi^{2}}\int _{0}^{T}\bigl\vert \dot{u}(t)\bigr\vert ^{2}\,dt \quad \text{(Wirtinger's inequality).} \end{aligned}$$

For the sake of convenience, we denote \(M_{1}=(\int_{0}^{T}f^{2}(t)\,dt)^{1/2}\), \(M_{2}=\int_{0}^{T}f(t)\,dt\), \(M_{3}=\int_{0}^{T}g(t)\,dt\).

Proof of Theorem 1.1

Due to (3), we can choose an \(a_{1}>T^{2}/(4\pi^{2})\) such that

$$ \liminf_{\vert x\vert \rightarrow\infty}h^{-2}\bigl(\vert x\vert \bigr)\int _{0}^{T}F(t,x) \,dt>\frac {a_{1}C_{0}^{2}}{2}M_{1}^{2}. $$
(2.1)

For (B) and the Sobolev inequality, for any \(u\in H_{T}^{1}\) we have

$$\begin{aligned} & \biggl\vert \int_{0}^{T} \bigl[F_{1} \bigl(t,u(t) \bigr)-F_{1}(t,\bar{u}) \bigr]\,dt \biggr\vert \\ &\quad = \biggl\vert \int_{0}^{T}\int _{0}^{1} \bigl(\nabla F_{1} \bigl(t,\bar {u}+s\tilde{u}(t) \bigr),\tilde{u}(t) \bigr)\,ds\,dt \biggr\vert \\ &\quad \leq\int_{0}^{T}\int_{0}^{1}f(t)h \bigl(\bigl\vert \bar{u}+s\tilde {u}(t)\bigr\vert \bigr)\bigl\vert \tilde{u}(t)\bigr\vert \,ds\,dt +\int_{0}^{T}\int _{0}^{1}g(t)\bigl\vert \tilde {u}(t)\bigr\vert \,ds\,dt \\ &\quad \leq\int_{0}^{T}\int _{0}^{1}C_{0}f(t) \bigl(h\bigl(\vert \bar {u}\vert \bigr)+h \bigl(\bigl\vert \tilde{u}(t)\bigr\vert \bigr) \bigr) \bigl\vert \widetilde{u}(t)\bigr\vert \,ds\,dt + M_{3}\Vert \tilde{u}\Vert _{\infty} \\ &\quad \leq C_{0}h\bigl(\vert \bar{u}\vert \bigr) \biggl(\int _{0}^{T}f^{2}(t) \,dt \biggr)^{1/2} \biggl(\int_{0}^{T}\bigl\vert \tilde{u}(t)\bigr\vert ^{2}\,dt \biggr)^{1/2} \\ &\qquad {}+ C_{0}\int_{0}^{T}f(t)h \bigl(\bigl\vert \tilde{u}(t)\bigr\vert \bigr)\bigl\vert \tilde {u}(t)\bigr\vert \,dt+M_{3}\Vert \tilde{u} \Vert _{\infty} \\ &\quad \leq C_{0}M_{1}h\bigl(\vert \bar{u}\vert \bigr)\Vert \tilde{u} \Vert _{L^{2}}+C_{0}\int_{0}^{T}f(t) \bigl(K_{1}\bigl\vert \tilde{u}(t)\bigr\vert ^{\alpha}+K_{2} \bigr)\bigl\vert \tilde{u}(t)\bigr\vert \,dt +M_{3}\Vert \tilde{u}\Vert _{\infty} \\ &\quad \leq C_{0}M_{1}h\bigl(\vert \bar{u}\vert \bigr)\Vert \tilde{u}\Vert _{L^{2}}+C_{0}M_{2}K_{1} \Vert \tilde{u} \Vert _{\infty}^{1+\alpha} +C_{0}M_{2}K_{2} \bigl\Vert \tilde{u}(t)\bigr\Vert _{\infty}+M_{3}\bigl\Vert \tilde{u}(t)\bigr\Vert _{\infty} \\ &\quad \leq\frac{1}{2a_{1}}\Vert \widetilde{u}\Vert ^{2}_{L^{2}}+ \frac {a_{1}(C_{0}M_{1})^{2}}{2}h^{2}\bigl(\vert \bar{u}\vert \bigr)+ C_{0}M_{2}K_{1} \Vert \tilde{u}\Vert _{\infty}^{1+\alpha} \\ &\qquad {}+C_{0}M_{2}K_{2} \bigl\Vert \tilde{u}(t)\bigr\Vert _{\infty}+M_{3}\bigl\Vert \tilde{u}(t)\bigr\Vert _{\infty} \\ &\quad \leq\frac{T^{2}}{8\pi^{2}a_{1}}\Vert \dot{u}\Vert _{L^{2}}^{2}+ \frac{a_{1}(C_{0}M_{1})^{2}}{2}h^{2}\bigl(\vert \bar{u}\vert \bigr) + \biggl( \frac{T}{12} \biggr)^{(1+\alpha)/2}C_{0}M_{2}K_{1} \Vert \dot{u}\Vert _{L^{2}}^{1+\alpha} \\ &\qquad {}+ \biggl(\frac{T}{12} \biggr)^{1/2}C_{0}M_{2}K_{2} \Vert \dot{u}\Vert _{L^{2}}+ \biggl(\frac{T}{12} \biggr)^{1/2}M_{3}\Vert \dot{u} \Vert _{L^{2}.} \end{aligned}$$
(2.2)

Similarly, from (2) and the Sobolev inequality, for any \(u\in H_{T}^{1} \) we get

$$\begin{aligned} &\int^{T}_{0} \bigl[F_{2} \bigl(u(t) \bigr)-F_{2}(\bar{u}) \bigr]\,dt \\ &\quad =\int_{0}^{T}\int_{0}^{1} \frac{1}{s} \bigl(\nabla F_{2} \bigl(\bar{u}+s\tilde {u}(t) \bigr)- \nabla F_{2}(\bar{u}),s\tilde{u}(t) \bigr)\,ds\,dt \\ &\quad \geq-\int_{0}^{T}\int _{0}^{1}rs^{\gamma-1}\bigl\vert \tilde {u}(t) \bigr\vert ^{\gamma}\,ds\,dt \\ &\quad \geq-\frac{rT}{\gamma} \Vert \widetilde{u}\Vert _{\infty}^{\gamma } \\ &\quad \geq-\frac{rT}{\gamma} \biggl(\frac{T}{12} \biggr)^{\gamma /2}\Vert \dot{u}\Vert _{L^{2}}^{\gamma}. \end{aligned}$$
(2.3)

From (2.2) and (2.3) we have

$$\begin{aligned} \varphi(u)={}&\frac{1}{2}\Vert \dot{u}\Vert _{L^{2}}^{2}+ \int_{0}^{T} \bigl[F_{1} \bigl(t,u(t) \bigr)-F_{1}(t,\bar{u}) \bigr]\,dt \\ &{} +\int^{T}_{0} \bigl[F_{2} \bigl(u(t) \bigr)-F_{2}(\bar{u}) \bigr]\,dt+\int_{0}^{T}F(t, \bar{u})\,dt \\ \geq {}&\biggl(\frac{1}{2}-\frac{T^{2}}{8\pi^{2}a_{1}} \biggr)\Vert \dot{u} \Vert _{L^{2}}^{2}-\frac{a_{1}(C_{0}M_{1})^{2}}{2}h^{2}\bigl(\vert \bar{u}\vert \bigr) - \biggl(\frac{T}{12} \biggr)^{\frac{1+\alpha}{2}}C_{0}M_{2}K_{1} \Vert \dot {u}\Vert _{L^{2}}^{1+\alpha} \\ &{} - \biggl(\frac{T}{12} \biggr)^{1/2}C_{0}M_{2}K_{2} \Vert \dot{u}\Vert _{L^{2}}- \biggl(\frac{T}{12} \biggr)^{1/2}M_{3} \Vert \dot{u}\Vert _{L^{2}} \\ &{}- \frac{rT}{\gamma} \biggl(\frac{T}{12} \biggr)^{\alpha/2}\Vert \dot{u} \Vert _{L^{2}}^{\gamma}+\int_{0}^{T}F(t, \bar{u})\,dt \\ \geq {}&\biggl(\frac{1}{2}-\frac{T^{2}}{8\pi^{2}a_{1}} \biggr)\Vert \dot{u} \Vert _{L^{2}}^{2} +h^{2}\bigl(\vert \bar{u}\vert \bigr) \biggl(h^{-2}\bigl(\vert \bar{u}\vert \bigr)\int _{0}^{T}F(t,\bar{u})\,dt- \frac {a_{1}(C_{0}M_{1})^{2}}{2} \biggr) \\ &{} - \biggl(\frac{T}{12} \biggr)^{1/2}(C_{0}M_{2}K_{2}+M_{3}) \Vert \dot{u}\Vert _{L^{2}}\\ &{} - \biggl(\frac{T}{12} \biggr)^{\frac{1+\alpha}{2}}C_{0}M_{2}K_{1} \Vert \dot {u}\Vert _{L^{2}}^{1+\alpha} -\frac{rT}{\gamma} \biggl( \frac{T}{12} \biggr)^{\gamma/2}\Vert \dot{u}\Vert _{L^{2}}^{\gamma}, \end{aligned}$$

for all \(u\in H_{T}^{1}\). So, by (2.1) we get \(\varphi(u)\rightarrow \infty\) as \(\Vert u\Vert \rightarrow\infty\).

Hence, applying the least action principle (see [6], Theorem 1.1 and Corollary 1.1), the proof is complete. □

Proof of Theorem 1.2

Step 1. First, we assert that φ satisfies the (PS) condition. Suppose that \(\{u_{n}\}\) is a (PS) sequence, that is, \(\varphi '(u_{n})\rightarrow0\) as \(n\rightarrow\infty\) and \(\{\varphi(u_{n})\} \) is bounded. For (5), we can choose an \(a_{2}>T^{2}/(4\pi^{2})\) such that

$$ \limsup_{\vert x\vert \rightarrow\infty} h^{-2}\bigl(\vert x\vert \bigr) \int_{0}^{T}F(t,x) \,dt < - \biggl( \frac{a_{2}}{2}+\frac{\sqrt{a_{2}}T}{2\pi} \biggr)C_{0}^{2} \int _{0}^{T}f^{2}(t)\,dt. $$
(2.4)

Similar to the proof of Theorem 1.1, we have

$$\begin{aligned} & \biggl\vert \int_{0}^{T} \bigl(\nabla F_{1} \bigl(t,u_{n}(t) \bigr),\tilde {u}_{n}(t) \bigr)\,dt \biggr\vert \\ & \quad \leq\frac{T^{2}}{8\pi^{2}a_{2}}\Vert \dot{u}_{n}\Vert _{L^{2}}^{2}+ \frac {a_{2}(C_{0}M_{1})^{2}}{2}h^{2}\bigl(\vert \bar{u}_{n}\vert \bigr) + \biggl( \frac{T}{12} \biggr)^{(1+\alpha)/2}C_{0}M_{2}K_{1} \Vert \dot{u}_{n}\Vert _{L^{2}}^{1+\alpha} \\ &\qquad {} + \biggl(\frac{T}{12} \biggr)^{1/2}(C_{0}M_{2}K_{2}+M_{3}) \Vert \dot{u}_{n}\Vert _{L^{2}} \end{aligned}$$
(2.5)

and

$$\begin{aligned} \int_{0}^{T} \bigl(\nabla F_{2} \bigl(u_{n}(t) \bigr),\tilde{u}_{n}(t) \bigr)\,dt \geq - \frac {rT}{\gamma} \biggl(\frac{T}{12} \biggr)^{\gamma/2}\Vert \dot{u} \Vert _{L^{2}}^{\gamma}, \end{aligned}$$

for all n. Hence we have

$$\begin{aligned} \Vert \tilde{u}_{n}\Vert \geq {}&\bigl\langle \varphi'(u_{n}), \tilde {u}_{n} \bigr\rangle \\ ={}&\Vert \dot{u}_{n}\Vert _{L^{2}}^{2}+\int _{0}^{T} \bigl(\nabla F \bigl(t,u_{n}(t) \bigr),\tilde{u}_{n}(t) \bigr)\,dt \\ \geq{}& \biggl(1- \frac{T^{2}}{8\pi^{2}a_{2}} \biggr)\Vert \dot {u}_{n} \Vert _{L^{2}}^{2}-\frac{a_{2}(C_{0}M_{1})^{2}}{2}h^{2}\bigl( \vert \bar{u}_{n}\vert \bigr) \\ &{}- \biggl(\frac{T}{12} \biggr)^{\frac{1+\alpha}{2}}C_{0}M_{2}K_{1} \Vert \dot {u}_{n}\Vert _{L^{2}}^{1+\alpha} \\ &{}- \biggl(\frac{T}{12} \biggr)^{1/2}(C_{0}M_{2}K_{2}+M_{3}) \Vert \dot{u}_{n}\Vert _{L^{2}} -\frac{rT}{\gamma} \biggl( \frac{T}{12} \biggr)^{\gamma/2}\Vert \dot{u}_{n}\Vert _{L^{2}}^{\gamma}, \end{aligned}$$
(2.6)

for large n. So, by Wirtinger’s inequality we get

$$ \bigl\Vert (\tilde{u}_{n})\bigr\Vert \leq\frac{ (T^{2}+4\pi^{2} )^{1/2}}{2\pi} \Vert \dot{u}_{n} \Vert _{L^{2}}. $$
(2.7)

From (2.6) and (2.7),

$$\begin{aligned} &\frac{a_{2}(C_{0}M_{1})^{2}}{2}h^{2}\bigl(\vert \bar{u}_{n}\vert \bigr) \\ &\quad \geq \biggl(1-\frac{T^{2}}{8\pi^{2}a_{2}} \biggr)\Vert \dot{u}_{n} \Vert _{L^{2}}^{2}- C_{0}M_{2}K_{1} \biggl(\frac{T}{12} \biggr)^{\frac{1+\alpha}{2}}\Vert \dot {u}_{n} \Vert _{L^{2}}^{1+\alpha} \\ &\qquad {}- \biggl(\frac{T}{12} \biggr)^{1/2}(C_{0}M_{2}K_{2}+M_{3}) \Vert \dot{u}_{n}\Vert _{L^{2}} \\ &\qquad {}-\Vert \tilde{u}_{n}\Vert - \frac{rT}{\gamma} \biggl( \frac{T}{12} \biggr)^{\gamma/2}\Vert \dot{u}_{n}\Vert _{L^{2}}^{\gamma} \geq\frac{1}{2}\Vert \dot{u}_{n} \Vert _{L^{2}}^{2}+C_{1}, \end{aligned}$$
(2.8)

where

$$\begin{aligned} C_{1}={}& \min_{s\in[0,+\infty]} \biggl\{ \frac{4\pi ^{2}a_{2}-T^{2}}{8\pi^{2}a_{2}}s^{2}- \biggl(\frac{T}{12} \biggr)^{\frac {1+\alpha}{2}}C_{0}M_{2}K_{1}s^{1+\alpha} \\ &{}- \biggl[\frac{ (T^{2}+4\pi^{2} )^{1/2}}{2\pi }+C_{0}M_{2}K_{2} \biggl(\frac{T}{12} \biggr)^{1/2}+ \biggl(\frac {T}{12} \biggr)^{1/2}M_{3} \biggr]s- \frac{rT}{\gamma} \biggl( \frac{T}{12} \biggr)^{\gamma/2}s^{\gamma} \biggr\} . \end{aligned}$$

Note that \(a_{2}>T^{2}/(4\pi^{2})\) implies \(-\infty < C_{1} <0\). Hence, it follows from (2.8) that

$$ \Vert \dot{u}_{n}\Vert _{L^{2}}^{2}\leq a_{2}C_{0}^{2}M_{1}^{2}h^{2} \bigl(\vert \overline {u}_{n}\vert \bigr)-2C_{1}, $$
(2.9)

and then

$$ \Vert \dot{u}_{n}\Vert _{L^{2}}\leq\sqrt{a_{2}}C_{0}M_{1}h \bigl(\vert \overline{u}_{n}\vert \bigr)+C_{2}, $$
(2.10)

where \(0 < C_{2} < +\infty\). Similar to the proof of Theorem 1.1, we have

$$\begin{aligned} & \biggl\vert \int_{0}^{T} \bigl[F_{1} \bigl(t,u_{n}(t) \bigr)-F_{1}(t,\bar {u}_{n}) \bigr] \,dt \biggr\vert \\ &\quad \leq C_{0}M_{1}h\bigl(\vert \bar{u}_{n} \vert \bigr)\Vert \tilde{u}_{n}\Vert _{L^{2}}+C_{0}M_{2}K_{1} \Vert \tilde{u}_{n}\Vert _{\infty}^{1+\alpha}+ (C_{0}M_{2}K_{2}+M_{3}) \Vert \tilde{u}_{n}\Vert _{\infty} \\ &\quad \leq\frac{\pi}{\sqrt{a_{2}}T}\Vert \tilde{u}_{n}\Vert _{L^{2}}^{2}+ \frac{\sqrt{a_{2}}TC_{0}^{2}}{4\pi}{M_{1}^{2}}h^{2} \bigl(\vert \bar{u}_{n}\vert \bigr) \\ &\qquad {} +C_{0}M_{2}K_{1} \Vert \tilde{u}_{n}\Vert _{\infty}^{1+\alpha }+(C_{0}M_{2}K_{2}+M_{3}) \Vert \tilde{u}_{n}\Vert _{\infty} \\ &\quad \leq\frac{T}{4\pi\sqrt{a_{2}}}\Vert \dot{u}_{n}\Vert _{L^{2}}^{2}+ \frac{\sqrt{a_{2}}TC_{0}^{2}}{4\pi}{M_{1}^{2}}h^{2} \bigl(\vert \bar{u}_{n}\vert \bigr) + \biggl(\frac{T}{12} \biggr)^{(1+\alpha)/2}C_{0}M_{2}K_{1} \Vert \dot{u}_{n}\Vert _{L^{2}}^{1+\alpha} \\ &\qquad {}+ \biggl(\frac{T}{12} \biggr)^{1/2}(C_{0}M_{2}K_{2}+M_{3}) \Vert \dot{u}_{n}\Vert _{L^{2}}. \end{aligned}$$
(2.11)

By (4), we obtain

$$\begin{aligned} &\int_{0}^{T} \bigl[F_{2} \bigl(u_{n}(t) \bigr)-F_{2}(\bar{u}_{n}) \bigr] \,dt \\ &\quad =\int_{0}^{T}\int_{0}^{1} \frac{1}{s} \bigl(\nabla F_{2} \bigl(\bar {u}_{n}+s \tilde{u}_{n}(t) \bigr)-\nabla F_{2}(\bar{u}_{n}),s \tilde{u}_{n}(t) \bigr)\,ds\,dt \\ &\quad \leq\int_{0}^{T}\int_{0}^{1} \mu s^{\delta-1}\bigl\vert \tilde {u}_{n}(t)\bigr\vert ^{\delta}\,ds\,dt \leq \frac{\mu T}{\delta} \Vert \tilde{u}_{n} \Vert _{\infty}^{\delta} \\ &\quad \leq\frac{\mu T}{\delta} \biggl(\frac{T}{12} \biggr)^{\delta/2}\Vert \dot{u}_{n}\Vert _{L^{2}}^{\delta}. \end{aligned}$$

From the boundedness of \({\varphi(u_{n})}\) and (2.9)-(2.11), we have

$$\begin{aligned} C_{3}\leq{}&\varphi(u_{n}) \\ ={}&\frac{1}{2}\Vert \dot {u}_{n}\Vert _{L^{2}}^{2} + \int_{0}^{T} \bigl[F_{1} \bigl(t,u_{n}(t) \bigr)-F_{1}(t,\bar{u}_{n}) \bigr] \,dt+ \int_{0}^{T} \bigl[F_{2} \bigl(u_{n}(t) \bigr)-F_{2}(\bar{u}_{n}) \bigr] \,dt \\ &{}+\int_{0}^{T}F \bigl(t,\bar{u}_{n}(t) \bigr)\,dt \\ \leq {}&\biggl(\frac{1}{2}+\frac{T}{4\pi\sqrt{a_{2}}} \biggr)\Vert \dot{u} \Vert _{L^{2}}^{2}+\frac{\sqrt{a_{2}}TC_{0}^{2}}{4\pi }{M_{1}^{2}}h^{2} \bigl(\vert \bar{u}\vert \bigr) +C_{0}M_{2}K_{1} \biggl(\frac{T}{12} \biggr)^{(1+\alpha)/2}\Vert \dot{u}_{n} \Vert _{L^{2}}^{1+\alpha} \\ &{}+ \biggl(\frac{T}{12} \biggr)^{1/2}(C_{0}M_{2}K_{2}+M_{3}) \Vert \dot{u}_{n}\Vert _{L^{2}} +\int_{0}^{T}F(t, \bar{u}_{n})\,dt+\frac{\mu T}{\delta} \biggl(\frac {T}{12} \biggr)^{\delta/2}\Vert \dot{u}_{n}\Vert _{L^{2}}^{\delta} \\ \leq {}&\biggl(\frac{1}{2}+\frac{T}{4\pi\sqrt{a_{2}}} \biggr) \bigl(a_{2}C_{0}^{2}M_{1}^{2}h^{2} \bigl(\vert \bar{u}_{n}\vert \bigr)-2C_{1} \bigr) + \frac{\sqrt{a_{2}}TC_{0}^{2}}{4\pi}{M_{1}^{2}}h^{2}\bigl(\vert \bar{u}\vert \bigr) \\ &{}+C_{0}M_{2}K_{1} \biggl(\frac{T}{12} \biggr)^{(1+\alpha )/2} \bigl(\sqrt{a_{2}}C_{0}M_{1}h \bigl(\vert \bar{u}_{n}\vert \bigr)+C_{2} \bigr)^{1+\alpha} \\ &{}+ \biggl(\frac{T}{12} \biggr)^{1/2}(C_{0}M_{2}K_{2}+M_{3}) \bigl(\sqrt{a_{2}}C_{0}M_{1}h\bigl(\vert \bar {u}_{n}\vert \bigr)+C_{2} \bigr) \\ &{}+ \int_{0}^{T}F(t,\bar{u}_{n})\,dt+ \frac{\mu T}{\delta} \biggl(\frac {T}{12} \biggr)^{\delta/2}\Vert \dot{u}_{n}\Vert _{L^{2}}^{\delta} \\ \leq {}&\biggl(\frac{a_{2}}{2}+\frac{\sqrt{a_{2}}T}{2\pi } \biggr)C_{0}^{2}M_{1}^{2}h^{2} \bigl(\vert \bar{u}_{n}\vert \bigr) - \biggl(1+\frac{T}{2\pi\sqrt{a_{2}}} \biggr)C_{1} \\ &{}+ \biggl(\frac{T}{12} \biggr)^{(1+\alpha)/2}2^{\alpha}C_{0}M_{2}K_{1} \bigl[ (\sqrt{a_{2}}C_{0}M_{1} )^{1+\alpha}h \bigl(\vert \bar {u}_{n}\vert \bigr)^{1+\alpha}+C_{2}^{1+\alpha} \bigr] \\ &{}+ \biggl(\frac{T}{12} \biggr)^{1/2}(C_{0}M_{2}K_{2}+M_{3}) \bigl(\sqrt{a_{2}}C_{0}M_{1}h\bigl(\vert \bar {u}_{n}\vert \bigr)+C_{2} \bigr) \\ &{}+\int_{0}^{T}F(t,\bar{u}_{n})\,dt + \frac{\mu T}{\delta} \biggl(\frac{T}{12} \biggr)^{\delta/2}2^{\delta -1} \bigl((\sqrt{a_{2}}M_{1})^{\delta}h^{\delta} \bigl(\vert \overline {u}_{n}\vert \bigr)+C_{2}^{\delta} \bigr) \\ ={}&h^{2}\bigl(\vert \bar{u}_{n}\vert \bigr) \biggl[h^{-2}\bigl(\vert \bar{u}_{n}\vert \bigr)\int _{0}^{T}F(t, \bar{u}_{n})\,dt + \biggl( \frac{a_{2}}{2}+\frac{\sqrt{a_{2}}T}{2\pi} \biggr)C_{0}^{2}M_{1}^{2} \\ &{}+ \biggl(\frac{T}{12} \biggr)^{(1+\alpha)/2}2^{\alpha }C_{0}^{2+\alpha}M_{2}M_{1}^{1+\alpha}K_{1}h^{\alpha-1} \bigl(\vert \bar{u}_{n}\vert \bigr) + \biggl(\frac{T}{12} \biggr)^{1/2}(C_{0}M_{2}K_{2} \\ &{}+M_{3}) \bigl(\sqrt{a_{2}}C_{0}M_{1}h^{-1} \bigl(\vert \bar{u}_{n}\vert \bigr) \bigr) +\frac{\mu T}{\delta} \biggl(\frac{T}{12} \biggr)^{\delta/2}2^{\delta -1}( \sqrt{a_{2}}M_{1})^{\delta}h^{\delta-2}\bigl(\vert \overline{u}_{n}\vert \bigr) \biggr] \\ &{}+ \biggl(\frac{T}{12} \biggr)^{(1+\alpha)/2}2^{\alpha }C_{0}M_{2}K_{1}C_{2}^{1+\alpha}+ \biggl(\frac{T}{12} \biggr)^{1/2}(C_{0}M_{2}K_{2} \\ &{}+M_{3})C_{2} - \biggl(1+\frac{T}{2\pi\sqrt{a_{2}}} \biggr)C_{1}+ \frac{\mu T}{\delta} \biggl(\frac{T}{12} \biggr)^{\delta/2}2^{\delta -1}C_{2}^{\delta}, \end{aligned}$$

for large n. So, by (2.4) we see that \(\vert \bar{u}\vert \) is bounded. Hence \(\{u_{n}\}\) is bounded by (2.9). Arguing as in the proof of Proposition 4.1 of [6], we conclude that the (PS) condition is satisfied.

Step 2. Let \(\tilde{H}_{T}^{1}=\{u\in H_{T}^{1}:\bar{u}=0\}\). We assert that for \(u\in\tilde{H}_{T}^{1}\),

$$ \varphi(u)\rightarrow+\infty,\qquad \Vert u\Vert \rightarrow\infty. $$
(2.12)

In fact, from (1) and Sobolev’s inequality, we get

$$\begin{aligned} & \biggl\vert \int_{0}^{T} \bigl[F_{1} \bigl(t,u(t) \bigr)-F_{1}(t,0) \bigr]\,dt \biggr\vert \\ &\quad = \biggl\vert \int_{0}^{T}\int _{0}^{1} \bigl(\nabla F \bigl(t,su(t) \bigr),u(t) \bigr)\,ds\,dt \biggr\vert \\ &\quad \leq\int_{0}^{T}f(t)h \bigl(\bigl\vert u(t) \bigr\vert \bigr)\bigl\vert u(t)\bigr\vert \,dt+\int_{0}^{T}g(t) \bigl\vert u(t)\bigr\vert \,dt \\ &\quad \leq\int_{0}^{T} f(t) \bigl(K_{1}\bigl\vert u(t)\bigr\vert ^{\alpha }+K_{2} \bigr)\bigl\vert u(t) \bigr\vert \,dt+M_{3}\Vert u\Vert _{\infty} \\ &\quad \leq M_{2}K_{1}\Vert u\Vert _{\infty}^{1+\alpha}+M_{2}K_{2} \Vert u\Vert _{\infty}+M_{3}\Vert u\Vert _{\infty} \\ &\quad \leq \biggl(\frac{T}{12} \biggr)^{\frac{1+\alpha }{2}}M_{2}K_{1} \Vert \dot{u}\Vert _{L^{2}}^{1+\alpha} + \biggl(\frac{T}{12} \biggr)^{1/2}(M_{2}K_{2}+M_{3})\Vert \dot{u} \Vert _{L^{2}}, \end{aligned}$$

for all \(u\in\tilde{H}_{T}^{1}\). It follows from (2) that

$$\begin{aligned} &\int^{T}_{0} \bigl[F_{2} \bigl(u(t) \bigr)-F_{2}(0) \bigr]\,dt \\ &\quad =\int_{0}^{T}\int_{0}^{1} \frac{1}{s} \bigl(\nabla F_{2} \bigl(s\tilde{u}(t) \bigr)-\nabla F_{2}(0),su(t) \bigr)\,ds\,dt \\ &\quad \geq-\int_{0}^{T}\int_{0}^{1}rs^{\gamma-1} \vert u\vert ^{\gamma } \,ds\,dt \\ &\quad \geq-\frac{rT}{\gamma} \Vert \dot{u}\Vert _{\infty}^{\gamma} \\ &\quad \geq-\frac{rT}{\gamma} \biggl(\frac{T}{12} \biggr)^{\gamma /2}\Vert \dot{u}\Vert _{L^{2}}^{\gamma}. \end{aligned}$$

So, we get

$$\begin{aligned} \varphi(u)={}&\frac{1}{2}\Vert \dot{u}\Vert _{L^{2}}^{2} +\int_{0}^{T} \bigl[F \bigl(t,u(t) \bigr)-F(t,0) \bigr]\,dt+ \int_{0}^{T}F(t,0)\,dt \\ \geq{}&\frac{1}{2}\Vert \dot{u}\Vert _{L^{2}}^{2}- \biggl( \frac {T}{12} \biggr)^{\frac{1+\alpha}{2}}M_{2}K_{1} \Vert \dot{u}\Vert _{L^{2}}^{1+\alpha} - \biggl(\frac{T}{12} \biggr)^{1/2}(M_{2}K_{2}+M_{3}) \Vert \dot{u}\Vert _{L^{2}} \\ &{}-\frac{rT}{\gamma} \biggl(\frac{T}{12} \biggr)^{\gamma/2}\Vert \dot{u} \Vert _{L^{2}}^{\gamma} +\int_{0}^{T}F(t,0) \,dt. \end{aligned}$$

By Wirtinger’s inequality, \(\Vert u\Vert \rightarrow \infty\) if and only if \(\Vert \dot{u}\Vert _{L^{2}} \rightarrow \infty\) in \(\tilde{H}_{T}^{1}\). Hence (2.12) holds.

Step 3. By (5), we can easily see that\(\int_{0}^{T}F(t,x)\,dt \rightarrow -\infty \) as \(\vert x\vert \rightarrow \infty\) for all \(x\in\mathbb{R}^{N}\). Thus, for all \(u\in (\tilde {H}_{T}^{1})^{\perp} =\mathbb{R}^{N}\),

$$\begin{aligned} \varphi(u)=\int_{0}^{T}F(t,u)\,dt \rightarrow- \infty \quad \text{as } \vert u\vert \rightarrow\infty. \end{aligned}$$

Now, by saddle point theorem (see, [10], Theorem 4.6), the proof is completed. □

Proof of Theorem 1.3

By (7), we can choose an \(a_{3}>\frac{T^{2}}{4\pi^{2}-rT^{2}}\) such that

$$ \liminf_{\vert x\vert \rightarrow\infty} h^{-2}\bigl(\vert x\vert \bigr) \int_{0}^{T}F(t,x) \,dt>\frac {a_{3}}{2}M_{1}^{2}C_{0}^{2}. $$
(2.13)

By (6) and the Sobolev inequality, we have

$$\begin{aligned} &\int_{0}^{T} \bigl[F_{2} \bigl(u(t) \bigr)-F_{2}(\bar{u}) \bigr]\,dt \\ &\quad =\int_{0}^{T}\int_{0}^{1} \frac{1}{s} \bigl(\nabla F_{2} \bigl(\bar{u}+s\tilde {u}(t) \bigr)- \nabla F_{2}(\bar{u}),s\tilde{u}(t) \bigr)\,ds\,dt \\ &\quad \geq -\int_{0}^{T}\int_{0}^{1}rs \bigl\vert \tilde{u}(t)\bigr\vert ^{2}\,ds\,dt \geq- \frac{rT^{2}}{8\pi^{2}}\Vert \dot{u} \Vert _{L^{2}}^{2}. \end{aligned}$$

By a similar method to that of the proof of Theorem 1.1, we get

$$\begin{aligned} \varphi(u)={}&\frac{1}{2}\Vert \dot{u}\Vert _{L^{2}}^{2} +\int_{0}^{T}F \bigl(t,u(t) \bigr)\,dt \\ ={}&\frac{1}{2}\Vert \dot{u}\Vert _{L^{2}}^{2} +\int _{0}^{T} \bigl[F_{1} \bigl(t,u(t) \bigr)-F_{1}(t,\overline{u}) \bigr]\,dt \\ &{}+\int_{0}^{T} \bigl[F_{2} \bigl(u(t) \bigr)-F_{2}(\overline{u}) \bigr]\,dt +\int_{0}^{T}F(t, \overline{u})\,dt \\ \geq {}&\biggl(\frac{1}{2}-\frac{T^{2}}{8\pi^{2}a_{3}}-\frac{rT^{2}}{8\pi ^{2}} \biggr) \Vert \dot{u}\Vert _{L^{2}}^{2} - \biggl(\frac{T}{12} \biggr)^{(1+\alpha)/2}C_{0}M_{2}K_{1}\Vert \dot{u}\Vert ^{1+\alpha}_{L_{2}} \\ &{}- \biggl(\frac{T}{12} \biggr)^{1/2} \biggl(M_{3}+ \frac {C_{0}M_{2}K_{2}}{2} \biggr)\Vert \dot{u}\Vert _{L^{2}} - \frac{a_{3}C_{0}^{2}M_{1}^{2}}{2}h^{2}\bigl(\vert \bar{u}\vert \bigr)+\int _{0}^{T}F(t,\overline{u})\,dt \\ ={}& \biggl(\frac{1}{2}-\frac{T^{2}}{8\pi^{2}a_{3}}-\frac {rT^{2}}{8\pi^{2}} \biggr)\Vert \dot{u}\Vert _{L^{2}}^{2} - \biggl(\frac{T}{12} \biggr)^{1/2} \biggl(M_{3}+\frac {C_{0}M_{2}K_{2}}{2} \biggr)\Vert \dot{u} \Vert _{L^{2}} \\ &{}- \biggl(\frac{T}{12} \biggr)^{(1+\alpha )/2}C_{0}M_{2}K_{1} \Vert \dot{u}\Vert ^{1+\alpha}_{L_{2}}+h^{2}\bigl( \vert \bar{u}\vert \bigr) \biggl(h^{-2}\bigl(\vert \bar{u}\vert \bigr)\int_{0}^{T}F(t, \bar{u})\,dt- \frac {a_{3}C_{0}^{2}M_{1}^{2}}{2} \biggr), \end{aligned}$$

for all \(u\in{H}_{T}^{1}\), which implies that \(\varphi(u)\rightarrow\infty\) as \(\Vert u\Vert \rightarrow\infty\) by (2.13), due to the facts that \(r<\frac{4\pi^{2}}{T^{2}}\) and \(\Vert u\Vert \rightarrow\infty\) if and only if \((\vert \bar{u}\vert ^{2}+\Vert \dot{u}\Vert _{L^{2}}^{2})^{1/2}\rightarrow\infty\). So, applying the least action principle, Theorem 1.3 holds. □

Proof of Theorem 1.4

First, we assert that φ satisfies the (PS) condition. Suppose that \(\{u_{n}\}\) satisfies \(\varphi'(u_{n})\rightarrow0\) as \(n\rightarrow\infty\) and \(\{\varphi (u_{n})\}\) is bounded. By (9), we can choose an \(a_{4}>\frac{T^{2}}{4\pi^{2}}\) such that

$$ \limsup_{\vert x\vert \rightarrow\infty} h^{-2}\bigl(\vert x\vert \bigr) \int_{0}^{T}F_{1}(t,x) \,dt< - \biggl( \frac{a_{4}}{2}+\frac{\sqrt{a_{4}}T}{2\pi } \biggr)C_{0}^{2}M_{1}^{2}. $$
(2.14)

By the (\(\lambda,\mu\))-subconvexity of \(G(x)\), we have

$$ G(x)\leq \bigl(2\mu \vert x\vert ^{\beta}+1 \bigr)G_{0} $$
(2.15)

for all \(x\in\mathbb{R}^{N}\), and a.e. \(t\in[0,T]\), where \(G_{0}=\max_{\vert s\vert \leq1}G(s)\), \(\beta=\log_{2\lambda}(2\mu)<2\) Then

$$\begin{aligned} &\int_{0}^{T} \bigl(\nabla F_{2} \bigl(t,u_{n}(t) \bigr),\tilde{u}_{n}(t) \bigr)\,dt \\ &\quad \geq-\int_{0}^{T}k(t)G(\bar{u}_{n}) \,dt \\ &\quad \geq-\int_{0}^{T}k(t) \bigl(2\mu \vert \bar{u}_{n}\vert ^{\beta}+1 \bigr)G_{0}\,dt \\ &\quad =-2\mu M_{4}\vert \bar{u}_{n}\vert ^{\beta}- M_{4}, \end{aligned}$$
(2.16)

where \(M_{4}=G_{0}\int_{0}^{T}k(t)\,dt\). From (2.5) and (2.16), for large n, we have

$$\begin{aligned} \Vert \bar{u}_{n}\Vert \geq {}&\bigl\langle \varphi(u_{n}), \tilde {u}_{n} \bigr\rangle \\ ={}&\Vert \dot{u}_{n}\Vert _{L^{2}}^{2}+\int _{0}^{T} \bigl(\nabla F \bigl(t,u_{n}(t) \bigr),\tilde{u}_{n}(t) \bigr) \\ \geq {}&\biggl(1-\frac{T^{2}}{8\pi^{2}a_{4}}\biggr)\Vert \dot{u}_{n} \Vert _{L^{2}}^{2}- \biggl( \frac{T}{12} \biggr)^{(1+\alpha )/2}C_{0}M_{2}K_{1} \Vert \dot{u}_{n}\Vert ^{1+\alpha}_{L^{2}} - \frac{(C_{0}M_{1})^{2}a_{4}}{2}h^{2}\bigl(\vert \bar{u}_{n}\vert \bigr) \\ &{}- \biggl(\frac{T}{12} \biggr)^{1/2} \biggl(M_{3}+ \frac {C_{0}M_{2}K_{2}}{2} \biggr)\Vert \dot{u}\Vert _{L^{2}} -2\mu M_{4}\vert \bar{u}_{n}\vert ^{\beta}- M_{4}. \end{aligned}$$
(2.17)

So, from (2.7) and (2.17) we have

$$\begin{aligned} &\frac{(C_{0}M_{1})^{2}a_{4}}{2}h^{2}\bigl(\vert \bar{u}_{n}\vert \bigr) +2\mu M_{4}\vert \bar{u}_{n}\vert ^{\beta} \\ &\quad \geq \biggl(1-\frac{T^{2}}{8\pi^{2}a_{4}} \biggr)\Vert \dot{u}_{n} \Vert _{L^{2}}^{2} - \biggl(\frac{T}{12} \biggr)^{(1+\alpha)/2}C_{0}M_{2}K_{1}\Vert \dot{u}_{n}\Vert ^{1+\alpha}_{L^{2}} \\ &\qquad {}- \biggl(\frac{T}{12} \biggr)^{1/2} \biggl(M_{3}+ \frac {C_{0}M_{2}K_{2}}{2} \biggr)\Vert \dot{u}\Vert _{L^{2}} - \frac{(T^{2}+4\pi^{2})^{1/2}}{2\pi} \Vert \dot{u}_{n}\Vert _{L^{2}}-M_{4} \\ &\quad \geq\frac{1}{2}\Vert \dot{u}_{n}\Vert ^{2}_{L^{2}}+C_{4}, \end{aligned}$$
(2.18)

where

$$\begin{aligned} C_{4}={}&\min \biggl\{ \biggl(\frac{1}{2}-\frac{T^{2}}{8\pi ^{2}a_{4}} \biggr)s^{2}- \biggl(\frac{T}{12} \biggr)^{(1+\alpha )/2}C_{0}M_{2}K_{1}s^{1+\alpha} - \biggl[\frac{ (T^{2}+4\pi^{2} )^{1/2}}{2\pi} \\ &{}+ \biggl(\frac{T}{12} \biggr)^{1/2} \biggl(M_{3}+ \frac{C_{0}M_{2}K_{2}}{2} \biggr) \biggr]s -M_{4} \biggr\} . \end{aligned}$$

Note that \(-\infty< C_{4}<0\) due to \(a_{4}>\frac{T^{2}}{4\pi^{2}}\), by (2.18), one has

$$ \Vert \dot{u}_{n}\Vert _{L^{2}}^{2}\leq a_{4}(C_{0}M_{1})^{2}h^{2} \bigl(\vert \bar{u}_{n}\vert \bigr)+4\mu M_{4}\vert \overline {u}_{n}\vert ^{\beta}-2C_{4}, $$
(2.19)

and then

$$ \Vert \dot{u}_{n}\Vert _{L^{2}}\leq \sqrt{a_{4}}C_{0}M_{1}h \bigl(\vert \bar{u}_{n}\vert \bigr)+2\sqrt{\mu M_{4}} \vert \overline {u}_{n}\vert ^{\beta/2}+C_{5}, $$
(2.20)

where \(C_{5} >0\). From (8) and (2.15), we have

$$\begin{aligned} & \biggl\vert \int_{0}^{T} \bigl[F_{2} \bigl(t,u(t) \bigr)-F_{2}(t,\bar{u}) \bigr]\,dt \biggr\vert \\ &\quad =\int_{0}^{T}\int_{0}^{1} \bigl(\nabla F_{2} \bigl(t,\bar {u}_{n}+s \tilde{u}_{n}(t) \bigr),\tilde{u}_{n}(t) \bigr)\,ds\,dt \\ &\quad \leq\int_{0}^{T}\int_{0}^{1}k(t)G \bigl(\bar {u}_{n}+(s+1)\tilde{u}_{n} \bigr)\,ds\,dt \\ &\quad \leq\int_{0}^{T}\int_{0}^{1}k(t) \bigl(2\mu\bigl\vert \bar {u}_{n}+(s+1)\tilde{u}_{n}(t) \bigr\vert ^{\beta}+1 \bigr) \\ &\quad \leq4\mu\int_{0}^{T}k(t) \bigl(\vert \bar{u}_{n}\vert ^{\beta}+2^{\beta }\vert \tilde{u}_{n}\vert ^{\beta} \bigr)G_{0}\int _{0}^{T}k(t)\,dt \\ &\quad \leq \biggl(\frac{T}{12} \biggr)^{\beta/2}2^{\beta+2}\mu M_{4}\Vert \dot{u}_{n}\Vert _{L^{2}}^{\beta} +4\mu M_{4}\vert \bar{u}_{n}\vert ^{\beta}+M_{4}, \end{aligned}$$
(2.21)

for all \(u\in H_{T}^{1}\). By the boundedness of \(\{\varphi(u_{n})\}\) and the inequalities (2.19)-(2.21), we get

$$\begin{aligned} C_{6}\leq{}&\varphi(u_{n}) \\ ={}&\frac{1}{2}\Vert \dot{u}_{n}\Vert _{L^{2}}^{2} +\int_{0}^{T} \bigl[F_{1} \bigl(t,u_{n}(t) \bigr)-F_{1}(t,\bar{u}_{n}) \bigr]\,dt \\ &{}+\int_{0}^{T} \bigl[F_{2} \bigl(t,u_{n}(t) \bigr)-F_{2}(t,\bar{u}_{n}) \bigr] \,dt +\int_{0}^{T}F(t,\bar{u}_{n}) \,dt \\ \leq {}&\biggl(\frac{1}{2}+\frac{T}{4\pi\sqrt{a_{4}}} \biggr)\Vert \dot{u}_{n} \Vert _{L^{2}}^{2}+\frac{\sqrt{a_{4}}TC_{0}^{2}}{4\pi}{M_{1}^{2}}h^{2} \bigl(\vert \bar{u}_{n}\vert \bigr) + \biggl(\frac{T}{12} \biggr)^{1/2}(C_{0}M_{2}K_{2}+M_{3}) \Vert \dot{u}_{n}\Vert _{L^{2}} \\ &{}+ \biggl(\frac{T}{12} \biggr)^{(1+\alpha )/2}C_{0}M_{2}K_{1} \Vert \dot{u}_{n}\Vert _{L^{2}}^{1+\alpha} + \biggl( \frac{T}{12} \biggr)^{\beta/2}2^{\beta+2}\mu M_{4} \Vert \dot{u}\Vert _{L^{2}}^{\beta}\\ &{}+4\mu M_{4}\vert \bar{u}_{n}\vert ^{\beta}+M_{4}+\int _{0}^{T}F(t, \bar{u}_{n})\,dt \\ \leq {}&\biggl(\frac{1}{2}+\frac{T}{4\pi\sqrt{a_{4}}} \biggr) \bigl(a_{4}(C_{0}M_{1})^{2}h^{2} \bigl(\vert \bar{u}_{n}\vert \bigr)+4\mu M_{4}\vert \overline {u}_{n}\vert ^{\beta}-2C_{4} \bigr)\\ &{}+ \frac{\sqrt{a_{4}}TC_{0}^{2}}{4\pi }{M_{1}^{2}}h^{2}\bigl(\vert \bar{u}_{n}\vert \bigr) \\ &{}+ \biggl(\frac{T}{12} \biggr)^{(1+\alpha )/2}C_{0}M_{2}K_{1} \bigl(\sqrt{a_{4}}C_{0}M_{1}h\bigl(\vert \bar{u}_{n}\vert \bigr)+2\sqrt {\mu M_{4}}\vert \overline{u}_{n}\vert ^{\beta/2}+C_{5} \bigr)^{1+\alpha} \\ &{}+ \biggl(\frac{T}{12} \biggr)^{1/2}(C_{0}M_{2}K_{2}+M_{3}) \bigl(\sqrt{a_{4}}C_{0}M_{1}h\bigl(\vert \bar {u}_{n}\vert \bigr)+2\sqrt{\mu M_{4}}\vert \overline{u}_{n}\vert ^{\beta/2}+C_{5} \bigr) \\ &{}+ \biggl(\frac{T}{12} \biggr)^{\beta/2}2^{\beta+2}\mu M_{4} \bigl(\sqrt{a_{4}}C_{0}M_{1}h \bigl(\vert \bar{u}_{n}\vert \bigr)+2\sqrt{\mu M_{4}} \vert \overline{u}_{n}\vert ^{\beta/2}+C_{5} \bigr)^{\beta}\\ &{}+4\mu M_{4}\vert \bar {u}_{n}\vert ^{\beta}+M_{4}+ \int_{0}^{T}F(t, \bar{u}_{n})\,dt \\ \leq {}&\biggl(\frac{a_{4}}{2}+\frac{\sqrt{a_{4}}T}{2\pi } \biggr) \bigl((C_{0}M_{1})^{2}h^{2} \bigl(\vert \bar{u}_{n}\vert \bigr) \bigr) \\ &{}+ \biggl(6+ \frac{T}{\pi\sqrt{a_{4}}} \biggr)\mu M_{4}\vert \overline {u}_{n} \vert ^{\beta}- \biggl(1+ \frac{T}{2\pi\sqrt{a_{4}}} \biggr)C_{4} \\ &{} + \biggl(\frac{T}{12} \biggr)^{(1+\alpha)/2}C_{0}M_{2}K_{1} \bigl(2^{\alpha }a_{4}^{(1+\alpha)/2}(C_{0}M_{1})^{1+\alpha}h^{1+\alpha} \bigl(\vert \overline{u}_{n}\vert \bigr) \\ &{}+2^{3\alpha+1} \mu^{\frac{1+\alpha}{2}}M_{4}^{\frac{1+\alpha }{2}}\vert \overline{u}_{n} \vert ^{\beta(1+\alpha)} +2^{2\alpha}C_{5}^{1+\alpha} \bigr) \\ &{}+ \biggl(\frac{T}{12} \biggr)^{\beta/2}2^{2+\beta}\mu M_{4} \bigl(2^{\beta-1}a_{4}^{\beta/2} \bigl(C_{0}M_{1}h\bigl(\vert \overline {u}_{n} \vert \bigr) \bigr)^{\beta}\\ &{}+2^{3\beta-2}\mu^{\beta/2}M_{4}^{\beta/2} \vert \overline{u}_{n}\vert ^{\beta^{2}/2}+2^{2(\beta-1)}C_{5}^{\beta} \bigr) \\ &{}+ \biggl(\frac{T}{12} \biggr)^{1/2} (C_{0}M_{2}K_{2}+M_{3} ) \bigl(\sqrt{a_{4}}C_{0}M_{1}h\bigl(\vert \bar{u}_{n}\vert \bigr)+2\sqrt{\mu M_{4}}\vert \overline {u}_{n}\vert ^{\beta/2}+C_{5} \bigr) \\ &{}+M_{4}+\int_{0}^{T}F_{1}(t, \bar{u}_{n})\,dt+\int_{0}^{T}F_{2}( \bar{u}_{n})\,dt \\ ={}&h^{2}\bigl(\vert \bar{u}_{n}\vert \bigr) \biggl[h^{-2}\bigl(\vert \bar{u}_{n}\vert \bigr)\int _{0}^{T}F_{1}(t, \bar{u}_{n}) \,dt+ \biggl(\frac{a_{4}}{2}+\frac{\sqrt{a_{4}}T}{2\pi} \biggr) (C_{0}M_{1})^{2} \\ &{}+ \biggl(\frac{T}{12} \biggr)^{(1+\alpha)/2}C_{0}^{2+\alpha }M_{1}^{1+\alpha}M_{2}K_{1}2^{\alpha}a_{4}^{(1+\alpha)/2}h^{\alpha -1} \bigl(\vert \overline{u}_{n}\vert \bigr) \\ &{}+ \biggl(\frac{T}{12} \biggr)^{\beta/2}2^{1+2\beta}\mu (C_{0}M_{1})^{\beta}M_{4}a_{4}^{\beta/2}h^{\beta-2} \bigl(\vert \overline {u}_{n}\vert \bigr)\\ &{}+ \biggl(\frac{T}{12} \biggr)^{1/2} (C_{0}M_{2}K_{2}+M_{3} ) \sqrt{a_{4}}C_{0}M_{1}h^{-1}\bigl( \vert \bar{u}_{n}\vert \bigr) \biggr] \\ &{}+\vert \overline{u}_{n}\vert ^{\beta} \biggl[\vert \overline{u}_{n}\vert ^{-\beta } \int_{0}^{T}F_{2}( \bar{u}_{n}) \,dt + \biggl(6+\frac{T}{\pi\sqrt{a_{4}}} \biggr)\mu M_{4}\\ &{}+ \biggl( \frac {T}{12} \biggr)^{(1+\alpha)/2}2^{3\alpha+1}C_{0}M_{2}K_{1} \mu^{(1+\alpha )/2}M_{4}^{(1+\alpha)/2}\vert \overline{u}_{n} \vert ^{\alpha\beta} \\ &{}+ \biggl(\frac{T}{12} \biggr)^{\beta/2}2^{4\beta}\mu ^{(\beta+2)/2}M_{4}^{(\beta+2)/2}\vert \overline{u}_{n} \vert ^{\frac{\beta ^{2}}{2}-\beta} \\ &{}+ \biggl(\frac{T}{12} \biggr)^{1/2}2 (C_{0}M_{2}K_{2}+M_{3} )\sqrt {\mu M_{4}}\vert \overline{u}_{n}\vert ^{-\beta/2} \biggr] \\ &{}- \biggl(1+\frac {T}{2\pi\sqrt{a_{4}}} \biggr)C_{4}+ \biggl(\frac{T}{12} \biggr)^{(1+\alpha )/2}2^{2\alpha}C_{0}M_{2}K_{1}C^{1+\alpha}_{5}\\ &{} + \biggl(\frac{T}{12} \biggr)^{1/2}(C_{0}M_{2}K_{2}+M_{3})C_{5} + \biggl(\frac{T}{12} \biggr)^{\beta/2}2^{3\beta}\mu M_{4}C^{\beta}_{5}+M_{4}, \end{aligned}$$

for large n. The above inequality and (2.14) imply that \(\{\vert \overline {u}\vert \}\) is bounded. Hence \(\{u_{n}\}\) is bounded by (2.19). By using the standard method, the (PS) condition holds.

Since the rest of the proof is similar to that of Theorem 1.2, we omit the details here. □

References

  1. Aizmahin, N, An, T: The existence of periodic solutions of non-autonomous second-order Hamiltonian systems. Nonlinear Anal. 74, 4862-4867 (2011)

    Article  MATH  MathSciNet  Google Scholar 

  2. Berger, MS, Schechter, M: On the solvability of semilinear gradient operator equations. Adv. Math. 25, 97-132 (1977)

    Article  MATH  MathSciNet  Google Scholar 

  3. Han, ZQ: 2π-periodic solutions to n-Duffing systems. In: Guo, DJ (ed.) Nonlinear Analysis and Its Applications, pp. 182-191. Beijng Scientific and Technical Publisher, Beijing (1994) (in Chinese)

    Google Scholar 

  4. Long, YM: Nonlinear oscillations for classical Hamiltonian systems with bi-even subquadratic potentials. Nonlinear Anal. 24, 1665-1671 (1995)

    Article  MATH  MathSciNet  Google Scholar 

  5. Mawhin, J: Semi-coercive monotone variational problems. Bull. Cl. Sci., Acad. R. Belg. 73, 118-130 (1987)

    MATH  MathSciNet  Google Scholar 

  6. Mawhin, J, Willem, M: Critical Point Theory and Hamiltonian Systems. Springer, New York (1989)

    Book  MATH  Google Scholar 

  7. Meng, Q, Tang, XH: Solutions of a second-order Hamiltonian system with periodic boundary conditions. Commun. Pure Appl. Anal. 9, 1053-1067 (2010)

    Article  MATH  MathSciNet  Google Scholar 

  8. Ma, J, Tang, CL: Periodic solutions for some nonautonomous second-order systems. J. Math. Anal. Appl. 275, 482-494 (2002)

    Article  MATH  MathSciNet  Google Scholar 

  9. Rabinowitz, PH: On subharmonic solutions of Hamiltonian systems. Commun. Pure Appl. Math. 33, 609-633 (1980)

    Article  MATH  MathSciNet  Google Scholar 

  10. Rabinowitz, PH: Minimax Methods in Critical Point with Applications to Differential Equations. CBMS, vol. 65. Am. Math. Soc., Providence (1986)

    Book  Google Scholar 

  11. Tang, CL: Periodic solutions of nonautonomous second order systems with γ-quasisubadditive potential. J. Math. Anal. Appl. 189, 671-675 (1995)

    Article  MATH  MathSciNet  Google Scholar 

  12. Tang, CL: Periodic solutions of nonautonomous second order systems. J. Math. Anal. Appl. 202, 465-469 (1996)

    Article  MATH  MathSciNet  Google Scholar 

  13. Tang, CL: Periodic solutions of nonautonomous second order systems with sublinear nonlinearity. Proc. Am. Math. Soc. 126, 3263-3270 (1998)

    Article  MATH  Google Scholar 

  14. Tang, CL, Wu, XP: Periodic solutions for second order systems with not uniformly coercive potentia. J. Math. Anal. Appl. 259, 386-397 (2001)

    Article  MATH  MathSciNet  Google Scholar 

  15. Tang, XH, Meng, Q: Solutions of a second-order Hamiltonian system with periodic boundary conditions. Nonlinear Anal., Real World Appl. 11, 3722-3733 (2010)

    Article  MATH  MathSciNet  Google Scholar 

  16. Willem, M: Oscillations forcées systèmes hamiltoniens. In: Public. Sémin. Analyse Non Linéaire. Université de Franche-Comté, Besancon (1981)

    Google Scholar 

  17. Wu, X: Saddle point characterization and multiplicity of periodic solutions of nonautonomous second order systems. Nonlinear Anal. 58, 899-907 (2004)

    Article  MATH  MathSciNet  Google Scholar 

  18. Wu, XP, Tang, CL: Periodic solutions of a class of nonautonomous second order systems. J. Math. Anal. Appl. 236, 227-235 (1999)

    Article  MATH  MathSciNet  Google Scholar 

  19. Zhao, F, Wu, X: Periodic solutions for a class of non-autonomous second order systems. J. Math. Anal. Appl. 296, 422-434 (2004)

    Article  MATH  MathSciNet  Google Scholar 

  20. Zhao, F, Wu, X: Existence and multiplicity of periodic solution for non-autonomous second-order systems with linear nonlinearity. Nonlinear Anal. 60, 325-335 (2005)

    MATH  MathSciNet  Google Scholar 

  21. Wang, ZY, Zhang, JH: Periodic solutions of a class of second order non-autonomous Hamiltonian systems. Nonlinear Anal. 72, 4480-4487 (2010)

    Article  MATH  MathSciNet  Google Scholar 

  22. Wu, Y, An, TQ: Existence of periodic solutions for non-autonomous second-order Hamiltonian systems. Electron. J. Differ. Equ. 77, 1 (2013)

    MathSciNet  Google Scholar 

Download references

Acknowledgements

The author thanks the referees and the editors for their helpful comments and suggestions. The research was supported by NSFC (11561043).

Author information

Authors and Affiliations

Authors

Corresponding author

Correspondence to Da-Bin Wang.

Additional information

Competing interests

The authors declare that they have no competing interests.

Authors’ contributions

The main idea of this paper was proposed by D-BW, D-BW prepared the manuscript initially, and KY performed a part of the steps of the proofs in this research. All authors read and approved the final manuscript.

Rights and permissions

Open Access This article is distributed under the terms of the Creative Commons Attribution 4.0 International License (http://creativecommons.org/licenses/by/4.0/), which permits unrestricted use, distribution, and reproduction in any medium, provided you give appropriate credit to the original author(s) and the source, provide a link to the Creative Commons license, and indicate if changes were made.

Reprints and permissions

About this article

Check for updates. Verify currency and authenticity via CrossMark

Cite this article

Wang, DB., Yang, K. Existence of periodic solutions for a class of second order Hamiltonian systems. Bound Value Probl 2015, 199 (2015). https://doi.org/10.1186/s13661-015-0460-z

Download citation

  • Received:

  • Accepted:

  • Published:

  • DOI: https://doi.org/10.1186/s13661-015-0460-z

Keywords